Indefinite Sum Extension of a Finite Sum Equality












0












$begingroup$


The other night I was considering the way in which we can split a finite sum of any arithmetic function into two finite sums, one for it's odd and another for even index terms :



$$sum _{k=1}^{n} left( -1 right) ^{k}a_{{k}}=sum _{k=1}^{ lfloor frac{n}{2} rfloor }a_{{2,k}}-sum _{k=0}^{lfloor frac{n}{2} rfloor -delta left( frac{n}{2},lfloor frac{n}{2} rfloor
right) +1}a_{{2,k+1}}
$$



$$quadquadquadquadquadquadquadquadquadquadquadquadquadquadquadquadquadquadquadquadquadquadquadquadquadquadquadquadquadquadquadquadquadquadquadoperatorname{(FS001)}$$



where:



$delta left( x,y right) =cases{1&$x=y$cr 0&$xneq y$cr}$
$$quadquadquadquadquadquadquadquadquadquadquadquadquadquadquadquadquadquadquadquadquadquadquadquadquadquadquadquadquadquadquadquadquadquadquadoperatorname{(KD001)}$$



And then I decided to be a little brave ask myself if this equality would hold for an infinite number of natural numbers:



$$lim _{nrightarrow infty}Bigl(sum _{k=1}^{n} left( -1 right) ^{k}a_{{k}}Bigr)=lim _{nrightarrow infty}Bigl(sum _{k=1}^{ lfloor frac{n}{2} rfloor }a_{{2,k}}Bigr)-lim _{nrightarrow infty}Bigl(sum _{k=0}^{lfloor frac{n}{2} rfloor -delta left( frac{n}{2},lfloor frac{n}{2} rfloor
right) +1}a_{{2,k+1}}
Bigr)$$



$$quadquadquadquadquadquadquadquadquadquadquadquadquadquadquadquadquadquadquadquadquadquadquadquadquadquadquadquadquadquadquadquadquadquadquadoperatorname{(IL001)}$$



Therefore the equality for the indefinite sum as follows:



$$sum _{k=1}^{infty} left( -1 right) ^{k}a_{{k}}=sum _{k=1}^{infty}a_{{2,k}}-sum _{k=0}^{infty}a_{{2,k+1}}
$$



$$quadquadquadquadquadquadquadquadquadquadquadquadquadquadquadquadquadquadquadquadquadquadquadquadquadquadquadquadquadquadquadquadquadquadquadoperatorname{(IL002)}$$



A preliminary investigation has lead to me holding a belief that this will be true in the event that $sum_{k=1}^{infty}a_{{k}}$ is a convergent sum, (therefore, as too are the sums on the right hand side of our equality above) however I have this unsettling feeling that even if for other arithmetic functions I use finite sum approximations to their divergent sums, and these results do indeed appear to confirm that the equality holds in the infinite limit, this will not always be the case and there will exist pathalogical residuals that are dependant on the specific arithmetic function in which they occur.



So my questions are:



1)Does this bad feeling I have about the indefinite extension of the equality have any merit to warrant further study?



2) If these non zero residual functions exist, would they serve as a good foundation for me to construct predicates a partition of the set of all arithmetic functions based on the nature of their from this equality?



Be nice please I don't like this subject very much.










share|cite|improve this question











$endgroup$

















    0












    $begingroup$


    The other night I was considering the way in which we can split a finite sum of any arithmetic function into two finite sums, one for it's odd and another for even index terms :



    $$sum _{k=1}^{n} left( -1 right) ^{k}a_{{k}}=sum _{k=1}^{ lfloor frac{n}{2} rfloor }a_{{2,k}}-sum _{k=0}^{lfloor frac{n}{2} rfloor -delta left( frac{n}{2},lfloor frac{n}{2} rfloor
    right) +1}a_{{2,k+1}}
    $$



    $$quadquadquadquadquadquadquadquadquadquadquadquadquadquadquadquadquadquadquadquadquadquadquadquadquadquadquadquadquadquadquadquadquadquadquadoperatorname{(FS001)}$$



    where:



    $delta left( x,y right) =cases{1&$x=y$cr 0&$xneq y$cr}$
    $$quadquadquadquadquadquadquadquadquadquadquadquadquadquadquadquadquadquadquadquadquadquadquadquadquadquadquadquadquadquadquadquadquadquadquadoperatorname{(KD001)}$$



    And then I decided to be a little brave ask myself if this equality would hold for an infinite number of natural numbers:



    $$lim _{nrightarrow infty}Bigl(sum _{k=1}^{n} left( -1 right) ^{k}a_{{k}}Bigr)=lim _{nrightarrow infty}Bigl(sum _{k=1}^{ lfloor frac{n}{2} rfloor }a_{{2,k}}Bigr)-lim _{nrightarrow infty}Bigl(sum _{k=0}^{lfloor frac{n}{2} rfloor -delta left( frac{n}{2},lfloor frac{n}{2} rfloor
    right) +1}a_{{2,k+1}}
    Bigr)$$



    $$quadquadquadquadquadquadquadquadquadquadquadquadquadquadquadquadquadquadquadquadquadquadquadquadquadquadquadquadquadquadquadquadquadquadquadoperatorname{(IL001)}$$



    Therefore the equality for the indefinite sum as follows:



    $$sum _{k=1}^{infty} left( -1 right) ^{k}a_{{k}}=sum _{k=1}^{infty}a_{{2,k}}-sum _{k=0}^{infty}a_{{2,k+1}}
    $$



    $$quadquadquadquadquadquadquadquadquadquadquadquadquadquadquadquadquadquadquadquadquadquadquadquadquadquadquadquadquadquadquadquadquadquadquadoperatorname{(IL002)}$$



    A preliminary investigation has lead to me holding a belief that this will be true in the event that $sum_{k=1}^{infty}a_{{k}}$ is a convergent sum, (therefore, as too are the sums on the right hand side of our equality above) however I have this unsettling feeling that even if for other arithmetic functions I use finite sum approximations to their divergent sums, and these results do indeed appear to confirm that the equality holds in the infinite limit, this will not always be the case and there will exist pathalogical residuals that are dependant on the specific arithmetic function in which they occur.



    So my questions are:



    1)Does this bad feeling I have about the indefinite extension of the equality have any merit to warrant further study?



    2) If these non zero residual functions exist, would they serve as a good foundation for me to construct predicates a partition of the set of all arithmetic functions based on the nature of their from this equality?



    Be nice please I don't like this subject very much.










    share|cite|improve this question











    $endgroup$















      0












      0








      0





      $begingroup$


      The other night I was considering the way in which we can split a finite sum of any arithmetic function into two finite sums, one for it's odd and another for even index terms :



      $$sum _{k=1}^{n} left( -1 right) ^{k}a_{{k}}=sum _{k=1}^{ lfloor frac{n}{2} rfloor }a_{{2,k}}-sum _{k=0}^{lfloor frac{n}{2} rfloor -delta left( frac{n}{2},lfloor frac{n}{2} rfloor
      right) +1}a_{{2,k+1}}
      $$



      $$quadquadquadquadquadquadquadquadquadquadquadquadquadquadquadquadquadquadquadquadquadquadquadquadquadquadquadquadquadquadquadquadquadquadquadoperatorname{(FS001)}$$



      where:



      $delta left( x,y right) =cases{1&$x=y$cr 0&$xneq y$cr}$
      $$quadquadquadquadquadquadquadquadquadquadquadquadquadquadquadquadquadquadquadquadquadquadquadquadquadquadquadquadquadquadquadquadquadquadquadoperatorname{(KD001)}$$



      And then I decided to be a little brave ask myself if this equality would hold for an infinite number of natural numbers:



      $$lim _{nrightarrow infty}Bigl(sum _{k=1}^{n} left( -1 right) ^{k}a_{{k}}Bigr)=lim _{nrightarrow infty}Bigl(sum _{k=1}^{ lfloor frac{n}{2} rfloor }a_{{2,k}}Bigr)-lim _{nrightarrow infty}Bigl(sum _{k=0}^{lfloor frac{n}{2} rfloor -delta left( frac{n}{2},lfloor frac{n}{2} rfloor
      right) +1}a_{{2,k+1}}
      Bigr)$$



      $$quadquadquadquadquadquadquadquadquadquadquadquadquadquadquadquadquadquadquadquadquadquadquadquadquadquadquadquadquadquadquadquadquadquadquadoperatorname{(IL001)}$$



      Therefore the equality for the indefinite sum as follows:



      $$sum _{k=1}^{infty} left( -1 right) ^{k}a_{{k}}=sum _{k=1}^{infty}a_{{2,k}}-sum _{k=0}^{infty}a_{{2,k+1}}
      $$



      $$quadquadquadquadquadquadquadquadquadquadquadquadquadquadquadquadquadquadquadquadquadquadquadquadquadquadquadquadquadquadquadquadquadquadquadoperatorname{(IL002)}$$



      A preliminary investigation has lead to me holding a belief that this will be true in the event that $sum_{k=1}^{infty}a_{{k}}$ is a convergent sum, (therefore, as too are the sums on the right hand side of our equality above) however I have this unsettling feeling that even if for other arithmetic functions I use finite sum approximations to their divergent sums, and these results do indeed appear to confirm that the equality holds in the infinite limit, this will not always be the case and there will exist pathalogical residuals that are dependant on the specific arithmetic function in which they occur.



      So my questions are:



      1)Does this bad feeling I have about the indefinite extension of the equality have any merit to warrant further study?



      2) If these non zero residual functions exist, would they serve as a good foundation for me to construct predicates a partition of the set of all arithmetic functions based on the nature of their from this equality?



      Be nice please I don't like this subject very much.










      share|cite|improve this question











      $endgroup$




      The other night I was considering the way in which we can split a finite sum of any arithmetic function into two finite sums, one for it's odd and another for even index terms :



      $$sum _{k=1}^{n} left( -1 right) ^{k}a_{{k}}=sum _{k=1}^{ lfloor frac{n}{2} rfloor }a_{{2,k}}-sum _{k=0}^{lfloor frac{n}{2} rfloor -delta left( frac{n}{2},lfloor frac{n}{2} rfloor
      right) +1}a_{{2,k+1}}
      $$



      $$quadquadquadquadquadquadquadquadquadquadquadquadquadquadquadquadquadquadquadquadquadquadquadquadquadquadquadquadquadquadquadquadquadquadquadoperatorname{(FS001)}$$



      where:



      $delta left( x,y right) =cases{1&$x=y$cr 0&$xneq y$cr}$
      $$quadquadquadquadquadquadquadquadquadquadquadquadquadquadquadquadquadquadquadquadquadquadquadquadquadquadquadquadquadquadquadquadquadquadquadoperatorname{(KD001)}$$



      And then I decided to be a little brave ask myself if this equality would hold for an infinite number of natural numbers:



      $$lim _{nrightarrow infty}Bigl(sum _{k=1}^{n} left( -1 right) ^{k}a_{{k}}Bigr)=lim _{nrightarrow infty}Bigl(sum _{k=1}^{ lfloor frac{n}{2} rfloor }a_{{2,k}}Bigr)-lim _{nrightarrow infty}Bigl(sum _{k=0}^{lfloor frac{n}{2} rfloor -delta left( frac{n}{2},lfloor frac{n}{2} rfloor
      right) +1}a_{{2,k+1}}
      Bigr)$$



      $$quadquadquadquadquadquadquadquadquadquadquadquadquadquadquadquadquadquadquadquadquadquadquadquadquadquadquadquadquadquadquadquadquadquadquadoperatorname{(IL001)}$$



      Therefore the equality for the indefinite sum as follows:



      $$sum _{k=1}^{infty} left( -1 right) ^{k}a_{{k}}=sum _{k=1}^{infty}a_{{2,k}}-sum _{k=0}^{infty}a_{{2,k+1}}
      $$



      $$quadquadquadquadquadquadquadquadquadquadquadquadquadquadquadquadquadquadquadquadquadquadquadquadquadquadquadquadquadquadquadquadquadquadquadoperatorname{(IL002)}$$



      A preliminary investigation has lead to me holding a belief that this will be true in the event that $sum_{k=1}^{infty}a_{{k}}$ is a convergent sum, (therefore, as too are the sums on the right hand side of our equality above) however I have this unsettling feeling that even if for other arithmetic functions I use finite sum approximations to their divergent sums, and these results do indeed appear to confirm that the equality holds in the infinite limit, this will not always be the case and there will exist pathalogical residuals that are dependant on the specific arithmetic function in which they occur.



      So my questions are:



      1)Does this bad feeling I have about the indefinite extension of the equality have any merit to warrant further study?



      2) If these non zero residual functions exist, would they serve as a good foundation for me to construct predicates a partition of the set of all arithmetic functions based on the nature of their from this equality?



      Be nice please I don't like this subject very much.







      real-analysis convergence special-functions divergent-series






      share|cite|improve this question















      share|cite|improve this question













      share|cite|improve this question




      share|cite|improve this question








      edited Dec 20 '18 at 11:14







      Adam

















      asked Dec 20 '18 at 10:15









      AdamAdam

      54114




      54114






















          1 Answer
          1






          active

          oldest

          votes


















          1












          $begingroup$

          If $a_k=(-1)^{k}frac 1 k$ then all the three series under consideration are divergent so the equation is not valid. Howoever, if $sum a_k$ and $sum (-1)^{k}a_k$ are both assumed to be convergent then everything is fine.






          share|cite|improve this answer











          $endgroup$













          • $begingroup$
            isn't $a_k$ divergent there? ah right the alternating does, ok you need to elaborate further are there any other examples that do not hold?
            $endgroup$
            – Adam
            Dec 20 '18 at 10:21












          • $begingroup$
            ok so that answers my first question, my concern was valid.
            $endgroup$
            – Adam
            Dec 20 '18 at 10:24










          • $begingroup$
            ok, but for the harmonic I need to understand why the finite sum equality fails in the infinite limit
            $endgroup$
            – Adam
            Dec 20 '18 at 10:26












          • $begingroup$
            Thankyou for your help Sir.
            $endgroup$
            – Adam
            Dec 20 '18 at 10:28











          Your Answer





          StackExchange.ifUsing("editor", function () {
          return StackExchange.using("mathjaxEditing", function () {
          StackExchange.MarkdownEditor.creationCallbacks.add(function (editor, postfix) {
          StackExchange.mathjaxEditing.prepareWmdForMathJax(editor, postfix, [["$", "$"], ["\\(","\\)"]]);
          });
          });
          }, "mathjax-editing");

          StackExchange.ready(function() {
          var channelOptions = {
          tags: "".split(" "),
          id: "69"
          };
          initTagRenderer("".split(" "), "".split(" "), channelOptions);

          StackExchange.using("externalEditor", function() {
          // Have to fire editor after snippets, if snippets enabled
          if (StackExchange.settings.snippets.snippetsEnabled) {
          StackExchange.using("snippets", function() {
          createEditor();
          });
          }
          else {
          createEditor();
          }
          });

          function createEditor() {
          StackExchange.prepareEditor({
          heartbeatType: 'answer',
          autoActivateHeartbeat: false,
          convertImagesToLinks: true,
          noModals: true,
          showLowRepImageUploadWarning: true,
          reputationToPostImages: 10,
          bindNavPrevention: true,
          postfix: "",
          imageUploader: {
          brandingHtml: "Powered by u003ca class="icon-imgur-white" href="https://imgur.com/"u003eu003c/au003e",
          contentPolicyHtml: "User contributions licensed under u003ca href="https://creativecommons.org/licenses/by-sa/3.0/"u003ecc by-sa 3.0 with attribution requiredu003c/au003e u003ca href="https://stackoverflow.com/legal/content-policy"u003e(content policy)u003c/au003e",
          allowUrls: true
          },
          noCode: true, onDemand: true,
          discardSelector: ".discard-answer"
          ,immediatelyShowMarkdownHelp:true
          });


          }
          });














          draft saved

          draft discarded


















          StackExchange.ready(
          function () {
          StackExchange.openid.initPostLogin('.new-post-login', 'https%3a%2f%2fmath.stackexchange.com%2fquestions%2f3047366%2findefinite-sum-extension-of-a-finite-sum-equality%23new-answer', 'question_page');
          }
          );

          Post as a guest















          Required, but never shown

























          1 Answer
          1






          active

          oldest

          votes








          1 Answer
          1






          active

          oldest

          votes









          active

          oldest

          votes






          active

          oldest

          votes









          1












          $begingroup$

          If $a_k=(-1)^{k}frac 1 k$ then all the three series under consideration are divergent so the equation is not valid. Howoever, if $sum a_k$ and $sum (-1)^{k}a_k$ are both assumed to be convergent then everything is fine.






          share|cite|improve this answer











          $endgroup$













          • $begingroup$
            isn't $a_k$ divergent there? ah right the alternating does, ok you need to elaborate further are there any other examples that do not hold?
            $endgroup$
            – Adam
            Dec 20 '18 at 10:21












          • $begingroup$
            ok so that answers my first question, my concern was valid.
            $endgroup$
            – Adam
            Dec 20 '18 at 10:24










          • $begingroup$
            ok, but for the harmonic I need to understand why the finite sum equality fails in the infinite limit
            $endgroup$
            – Adam
            Dec 20 '18 at 10:26












          • $begingroup$
            Thankyou for your help Sir.
            $endgroup$
            – Adam
            Dec 20 '18 at 10:28
















          1












          $begingroup$

          If $a_k=(-1)^{k}frac 1 k$ then all the three series under consideration are divergent so the equation is not valid. Howoever, if $sum a_k$ and $sum (-1)^{k}a_k$ are both assumed to be convergent then everything is fine.






          share|cite|improve this answer











          $endgroup$













          • $begingroup$
            isn't $a_k$ divergent there? ah right the alternating does, ok you need to elaborate further are there any other examples that do not hold?
            $endgroup$
            – Adam
            Dec 20 '18 at 10:21












          • $begingroup$
            ok so that answers my first question, my concern was valid.
            $endgroup$
            – Adam
            Dec 20 '18 at 10:24










          • $begingroup$
            ok, but for the harmonic I need to understand why the finite sum equality fails in the infinite limit
            $endgroup$
            – Adam
            Dec 20 '18 at 10:26












          • $begingroup$
            Thankyou for your help Sir.
            $endgroup$
            – Adam
            Dec 20 '18 at 10:28














          1












          1








          1





          $begingroup$

          If $a_k=(-1)^{k}frac 1 k$ then all the three series under consideration are divergent so the equation is not valid. Howoever, if $sum a_k$ and $sum (-1)^{k}a_k$ are both assumed to be convergent then everything is fine.






          share|cite|improve this answer











          $endgroup$



          If $a_k=(-1)^{k}frac 1 k$ then all the three series under consideration are divergent so the equation is not valid. Howoever, if $sum a_k$ and $sum (-1)^{k}a_k$ are both assumed to be convergent then everything is fine.







          share|cite|improve this answer














          share|cite|improve this answer



          share|cite|improve this answer








          edited Dec 20 '18 at 10:25

























          answered Dec 20 '18 at 10:20









          Kavi Rama MurthyKavi Rama Murthy

          60.6k42161




          60.6k42161












          • $begingroup$
            isn't $a_k$ divergent there? ah right the alternating does, ok you need to elaborate further are there any other examples that do not hold?
            $endgroup$
            – Adam
            Dec 20 '18 at 10:21












          • $begingroup$
            ok so that answers my first question, my concern was valid.
            $endgroup$
            – Adam
            Dec 20 '18 at 10:24










          • $begingroup$
            ok, but for the harmonic I need to understand why the finite sum equality fails in the infinite limit
            $endgroup$
            – Adam
            Dec 20 '18 at 10:26












          • $begingroup$
            Thankyou for your help Sir.
            $endgroup$
            – Adam
            Dec 20 '18 at 10:28


















          • $begingroup$
            isn't $a_k$ divergent there? ah right the alternating does, ok you need to elaborate further are there any other examples that do not hold?
            $endgroup$
            – Adam
            Dec 20 '18 at 10:21












          • $begingroup$
            ok so that answers my first question, my concern was valid.
            $endgroup$
            – Adam
            Dec 20 '18 at 10:24










          • $begingroup$
            ok, but for the harmonic I need to understand why the finite sum equality fails in the infinite limit
            $endgroup$
            – Adam
            Dec 20 '18 at 10:26












          • $begingroup$
            Thankyou for your help Sir.
            $endgroup$
            – Adam
            Dec 20 '18 at 10:28
















          $begingroup$
          isn't $a_k$ divergent there? ah right the alternating does, ok you need to elaborate further are there any other examples that do not hold?
          $endgroup$
          – Adam
          Dec 20 '18 at 10:21






          $begingroup$
          isn't $a_k$ divergent there? ah right the alternating does, ok you need to elaborate further are there any other examples that do not hold?
          $endgroup$
          – Adam
          Dec 20 '18 at 10:21














          $begingroup$
          ok so that answers my first question, my concern was valid.
          $endgroup$
          – Adam
          Dec 20 '18 at 10:24




          $begingroup$
          ok so that answers my first question, my concern was valid.
          $endgroup$
          – Adam
          Dec 20 '18 at 10:24












          $begingroup$
          ok, but for the harmonic I need to understand why the finite sum equality fails in the infinite limit
          $endgroup$
          – Adam
          Dec 20 '18 at 10:26






          $begingroup$
          ok, but for the harmonic I need to understand why the finite sum equality fails in the infinite limit
          $endgroup$
          – Adam
          Dec 20 '18 at 10:26














          $begingroup$
          Thankyou for your help Sir.
          $endgroup$
          – Adam
          Dec 20 '18 at 10:28




          $begingroup$
          Thankyou for your help Sir.
          $endgroup$
          – Adam
          Dec 20 '18 at 10:28


















          draft saved

          draft discarded




















































          Thanks for contributing an answer to Mathematics Stack Exchange!


          • Please be sure to answer the question. Provide details and share your research!

          But avoid



          • Asking for help, clarification, or responding to other answers.

          • Making statements based on opinion; back them up with references or personal experience.


          Use MathJax to format equations. MathJax reference.


          To learn more, see our tips on writing great answers.




          draft saved


          draft discarded














          StackExchange.ready(
          function () {
          StackExchange.openid.initPostLogin('.new-post-login', 'https%3a%2f%2fmath.stackexchange.com%2fquestions%2f3047366%2findefinite-sum-extension-of-a-finite-sum-equality%23new-answer', 'question_page');
          }
          );

          Post as a guest















          Required, but never shown





















































          Required, but never shown














          Required, but never shown












          Required, but never shown







          Required, but never shown

































          Required, but never shown














          Required, but never shown












          Required, but never shown







          Required, but never shown







          Popular posts from this blog

          Quarter-circle Tiles

          build a pushdown automaton that recognizes the reverse language of a given pushdown automaton?

          Mont Emei